Download as pdf or txt
Download as pdf or txt
You are on page 1of 53

1. Consider the following statements with 2.

Consider the following statements with


reference to Composition of atmosphere: reference to Exosphere:
1. Nitrogen and oxygen make up nearly 1. Light gases like helium and hydrogen
99% of the clean, dry air. float into space from here.
2. Nitrogen is important for controlling 2. Temperature gradually increases
combustion by diluting oxygen. through the layer.
Which of the above statements is/are 3. This layer coincides with space.
correct?
Which of the above statements is/are
A. 1 only correct?
B. 2 only A. 1 and 2 only
C. Both 1 and 2 B. 2 and 3 only
D. Neither 1 nor 2 C. 1 and 3 only
Answer: C D. 1, 2 and 3
Explanation: Answer: D
Statement 1 is correct: Explanation:
• Nitrogen and oxygen make up nearly Statement 1 is correct: Light gases like helium
99% of the clean, dry air. The and hydrogen float into space from here.
remaining gases are mostly inert and Statement 2 is correct: Temperature gradually
constitute about 1% of the atmosphere. increases through the layer.
Statement 3 is correct: This layer coincides
• Besides these gases, large quantities of
with space.
water vapour and dust particles are
also present in the atmosphere. These Exosphere
solid and liquid particles are of great
• This is the uppermost layer of the
climatic significance.
atmosphere extending beyond the
Statement 2 is correct: ionosphere above a height of about 400
km.
• Nitrogen is important for controlling
combustion by diluting oxygen. • The air is extremely rarefied and the
Nitrogen accounts for 78% of total temperature gradually increases
atmospheric volume. through the layer.
• It is a relatively inert gas, and is an • Light gases like helium and hydrogen
important constituent of all organic float into space from here.
compounds. It also indirectly helps in
• Temperature gradually increases
oxidation of different kinds.
through the layer. (As it is exposed to
direct sunlight)
• This layer coincides with space. of radiation takes place.
• If the wavelength is less than the
3. Which of the following Factors Affect obstructing particle (such as a dust
Temperature Distribution? particle), then total reflection takes
place.
1. The Angle of Incidence
• Absorption of solar radiation takes
2. Transparency of Atmosphere
place if the obstructing particles
3. Prevailing Winds happen to be water vapour, ozone
4. Earth’s Distance from Sun molecules, carbon dioxide molecules
or clouds.
Select the correct answer using the codes
given below: • Most of the light received by earth is
scattered light.
A. 1 and 2 only
Statement 3 is correct:
B. 3 only
• Prevailing Winds transfer heat from
C. 1 and 4 only
one latitude to another. They also help
D. 1,2,3 and 4 in exchange of heat between land and
Answer: D water bodies.

Explanation: • The oceanic winds have the capacity to


take the moderating influence of the
Statement 1 is correct:
sea to coastal areas – reflected in cool
• The angle of incidenceThe angle of summers and mild winters. This effect
incidence of the sun's rays decreases is pronounced only on the windward
from the equator to the poles. side (the side facing the ocean).
• The temperature rises as the angle of • The leeward side or the interiors do not
incidence rises. In the same way, the get the moderating effect of the sea,
lower the angle of incidence, the lower and therefore experience extremes of
the temperature. temperature
• As a result, temperatures are greater Statement 4 is correct:
towards the tropics and lower towards • Earth’s Distance from Sun During its
the poles. revolution around the sun, the earth is
Statement 2 is correct: farthest from the sun (152 million km
on 4th July). This position of the earth
• Transparency of AtmosphereAerosols
is called.
(smoke, sooth), dust, water vapor,
clouds etc. affect transparency. • On 3rd January, the earth is the nearest
to the sun (147 million km). This
• If the wavelength (X) of the radiation is
position is called.
more than the radius of the obstructing
particle (such as a gas), then scattering • Therefore, the annual insolation
received by the earth on 3rd January is the factors that cause these variations:
slightly more than the amount
• The rotation of the earth on its axis.
received on 4th July.
• The angle of inclination of the rays of
• However, the effect of this variation in
the sun
the solar output is masked by other
factors like the distribution of land and • The length of the day.
sea and the atmospheric circulation. • The transparency of the atmosphere,
• Hence, this variation in the solar and
output does not have a great effect on • The configuration of the land in terms
daily weather changes on the surface of its aspect.
of the earth.
• (The insolation depends more on the
first three factors)
4. The amount and intensity of solar Additional information:
radiation received by the earth (insolation)
The tilted position of the earth’s axis is known
vary during a day, in a season and in a year.
as the inclination of the earth’s axis. The
The following are the factors that cause
earth’s rotation axis makes an angle of about
these variations:
66.5° with the plane of its orbit around the sun
1. The rotation of the earth on its axis. and this greatly influences the amount of
2. The angle of inclination of the rays of insolation received at different places.
the sun The amount of insolation also depends on the
3. The length of the day. angle of inclination of the sun’s rays. The
higher the latitude the less is the angle they
4. The transparency of the atmosphere,
make with the surface of the earth which
5. The configuration of the land in terms results in slant sun rays.
of its aspect.
• The slant rays cover more area than the
Select the correct answer using the codes vertical rays. When more area is
given below: covered, the energy gets distributed
A. 1 and 2 only and the net energy received per area
decreases. Also, the slant rays have to
B. 3, 4 and 5 only
pass through a greater depth of the
C. 3 and 5 only atmosphere which results in more
D. 1,2,3,4 and 5 absorption, diffusion and scattering.

Answer: D • Before striking the earth’s surface, the


Explanation: incoming solar radiation passes
through the atmosphere. The
• The amount and intensity of solar atmosphere is largely transparent to
radiation received by the earth shortwave solar radiation.
(insolation) vary during a day, in a
season and in a year. The following are • Water vapors, ozone and other gases
present in the atmosphere absorb most D. Neither 1 nor 2
of the near-infrared radiations. Small Answer: C
suspended particles in the troposphere
scatter the visible spectrum both to Explanation:
space and towards the surface of the Statement 1 is correct: The surplus heat
earth. energy from the tropics is redistributed pole
wards and as a result the tropics do not get
• The blue colour of the sky and the red
progressively heated up due to the
colour of the rising and setting sun are
accumulation of excess heat or the high
the results of the scattering of light
latitudes get permanently frozen due to
within the atmosphere.
excess deficit.
Duration of the day varies from place
to place and season to season. It Statement 2 is correct: When 100% of solar
decides the amount of insolation radiation reaches the earth’s atmosphere,
received on the earth’s surface. about 35% is reflected back to space even
before reaching the surface of the earth. The
• The amount of solar radiation received
reflected amount is called the albedo of the
at the surface of the earth is more in the
earth. This amount of energy does not heat
tropics (about 320 watts/m²) and least
either the earth or the atmosphere.
in the poles (70 watts/m²). The
subtropical deserts receive maximum
insolation as the atmosphere is more 6. Consider the following statements with
transparent (least cloudiness). At the reference to Sub-Polar Low-Pressure Belt:
same latitude, the insolation is more
1. During winter one of the distinct low
over the continents than over the
centre of the belt is between Iceland
oceans.
and Greenland
2. Polar jet streams are produced here.
5. Consider the following statements with
Which of the above statements is/are
respect to Heat budget of the Earth:
correct?
1. The surplus heat energy from the
A. 1 only
tropics is redistributed towards the
pole. B. 2 only

2. Roughly 35% of insolation are reflected C. Both 1 and 2


back to space even before reaching the D. Neither 1 nor 2
earth’s surface
Answer: C
Which of the statements given above is/are
Explanation:
correct?
Statement 1 is correct: During winter, because of a
A. 1 only
high contrast between land and sea, this belt is
B. 2 only broken into two distinct low centers – one in the
C. Both 1 and 2
vicinity of the Aleutian Islands and the other reversal of winds from northeast to
between Iceland and Greenland. south and southwest, takes place. They
are called northeast monsoons.
Statement 2 is correct: During summer, a lesser
contrast results in a more developed and regular
belt. The area of contrast between cold and warm 8. Which of the following belts is called
air masses produces polar jet streams which doldrums?
encircles the earth at 60 degrees latitudes and is
A. Equatorial Low-Pressure Belt
focused in these low-pressure areas.
B. Sub-tropical High-Pressure Belts

7. Consider the following statements with C. Sub-polar low-Pressure Belts


reference to the Inter-Tropical Convergence D. Polar High-Pressure Belts
Zone (ITCZ):
Answer: A
1. It is a zone where air tends to descend
Explanation:
2. In July, the ITCZ is located around the
• The Equatorial Low-Pressure Belt:
monsoon trough.
• The sun shines almost vertically on the
3. In winter, the ITCZ moves southward
equator throughout the year.
Which of the above statements is/are
• As a result, the air gets warm and rises
correct?
over the equatorial region and
A. 1 only produces equatorial low pressure.
B. 2 and 3 only This belt extends from the equator to
10º N and 10 º S latitudes.
C. 3 only
• Due to excessive heating horizontal
D. 1 and 2 only
movement of air is absent here and
Answer: B only conventional currents are there.
Explanation: • Therefore, this belt is called doldrums
• Statement 1 is incorrect: The Inter (the zone of calm) due to virtual
Tropical Convergence Zone (ITCZ) is a absence of surface winds.
low-pressure zone located at the • These are the regions of convergence
equator where trade winds converge, because the winds flowing from sub-
and so, it is a zone where air tends to tropical high-pressure belts converge
ascend. here.
• Statement 2 is correct: In July, the • This belt is also known as-Inter
ITCZ is located around 20°N-25°N Tropical Convergence Zone (ITCZ).
latitudes (over the Gangetic plain),
sometimes called the monsoon trough.
9. Consider the following statements:
• Statement 3 is correct: In winter, the
ITCZ moves southward, and so the 1. The velocity of winds is related to the
direction in which they blow the equator (known as the trade
winds).
2. In the northern hemisphere, the wind
blowing from the Horse latitudes to • These diverging winds are the result of
the Doldrums is called Trade wind an area of high pressure, which is
Which of the above statements is/are characterized by calm winds, sunny
correct? skies, and little or no precipitation.

A. 1 only
B. 2 only 10. Consider the following statements:

C. Both 1 and 2 1. Geostrophic flow result from a balance


between the Coriolis force and the
D. Neither 1 nor 2
pressure gradient force
Answer: B
2. The wind movement around a low is
Explanation: called anti-cyclonic circulation.
Statement 1 s incorrect: The velocity of winds
Which of the above statements is/are
is related to Pressure gradient in the direction
correct?
of their flow.
A. 1 only
Statement 2 is correct: In the northern
hemisphere, the wind blowing from the B. 2 only
Horse latitudes to the Doldrums is called C. Both 1 and 2
Trade wind.
D. Neither 1 nor 2
Additional Information:
Answer: A
• The horse latitudes are subtropical
Explanation:
regions known for calm winds and
little precipitation. Statement 1 is correct: Geostrophic flow result
from a balance between the Coriolis force and
the pressure gradient force
Statement 2 is incorrect: The wind movement
around a low is called cyclonic circulation.
Around a high it is called anti-cyclonic
circulation. The direction of winds around
such systems changes according to their
• The horse latitudes are located at about
location in different hemispheres.
30 degrees north and south of the
equator.
• It is common in this region of the 11. Consider the statements regarding the
subtropics for winds to diverge and distribution of temperature on the earth
either flow toward the poles (known as surface:
the prevailing westerlies) or toward 1. Highest temperature is recorded on
the equator. D. All of the above
2. Low temperature is recorded on the Answer: D
adret side of the slope. Explanation:
3. With increase in height the Factors Affecting Temperature Distribution:
temperature decreases.
• Latitude: The temperature of the
Select the correct answer using the codes surface water decreases from the
given below: equator towards the poles because the
A. 3 only sun rays become more and more
inclined and hence the amount of
B. 1 and 2 only
insolation minimizes poleward.
C. 1 and 3 only
• Unequal Distribution of Land and
D. 1, 2 and 3 Water: The oceans in the northern
Answer: A hemisphere receive more heat because
Explanation: of their contact with the larger extent of
land than the equivalent parts in the
Statement 1 is incorrect: Maximum southern hemisphere.
temperature is not recorded on equator but
rather than on 20 degree north. • Prevailing Winds: The winds blowing
from the land towards the ocean drive
Statement 2 is incorrect: High temperature is
surface water away from the coasts
recorded on adret (sun side facing) slope.
resulting in an upwelling, in which
Statement 3 is correct: With increase in height deep cold water rises into the surface.
the temperature decreases due to normal
• Ocean Current: Warm Ocean current
lapse rate
increases the temperature in cold areas
whereas the cold current decreases the
12. Factors affecting the temperature of a temperature in the warm ocean. For
place on the earth surface are: example: in a gulf stream, a warmer
current increase the temperature of the
1. latitude of the place
Eastern coast of North America and
2. altitude of place the west coast of Europe.
3. nature of ground surface or slope • The Altitude: The atmosphere is
4. distance from the sea mostly heated by terrestrial radiation.
Therefore, the places near the earth's
5. prevailing winds
surface are warmer than the places
Select the correct answer using the codes away from the surface (at higher
given below: elevations).Hence, as the height
A. 1, 2 and 3 only increases from the earth surface, the
temperature goes on decreasing.
B. 1, 2 and 4 only
Nature of surface:
C. 1, 3, 4 and 5 only
Select the correct answer using the codes
given below:
A. 1>3>4>2>5
B. 2>3>5>1>4
C. 1>4>3>2>5
D. 4>2>3>5>1
Answer: C

14. Consider the following statements with


• Other factors affecting the temperature reference to the temperature inversion:
distribution are local weather 1. It means increase in temperature with
conditions like storms and cyclones. increase in height
2. Mountain and valley breezes are
13. Arrange the following surfaces in examples of temperature inversion.
decreasing order of their albedo (reflection 3. Static air and snow-covered surfaces
of incoming solar radiations) are favourable for temperature
1. snow / ice covered surfaces inversion.

2. thickly covered vegetation Which of the statements given above are


correct?
3. Grasslands
A. 1 and 2 only
4. barren / desert land
B. 2 and 3 only
5. ocean water
C. 1 and 3 only
D. 1, 2 and 3 only B. 2 only
Answer: B C. Both 1 and 2
D. Neither 1 nor 2
Answer: C
Explanation:
Statement 1 and 2 are correct:
Favorable Conditions for Temperature
Inversion are:
• Long winter nights
• Cloudless and clear sky
• Air drainage down the slope of the
mountains.
• Dry air near the ground surface
• Slow movement of air that reduces
mixing of heat in the lower layers of
the atmosphere.
• Ground surface covered with snow
that reflects insolation.

15. Which of the following conditions may


lead to the inversion of temperature?
16. Match the following:
1. A long winter night with cloudless
Sky. List 1 List 2

2. Air drainage down the slope of the A. Temperature falls 1. Ionosphere


mountains. with height
Select the correct answer using the code
B. Reflects radio waves 2. Stratosphere
given below:
back to the Earth
A. 1 only
causing the water vapour to condense.
C. Contains most of the 3. Tropopause
ozone • This condensed vapour is dew.

D. Fall in temperature 4. Troposphere • On a cloudy night, the clouds send the


stops heat back to the ground so the ground
never gets cold enough for the dew to
Select the correct answer using the codes be formed.
given below:
• The dew formation is more when the
A. 4 2 1 3 sky is clear and less when it is cloudy.
B. 3 2 4 1 • When the sky is clear and the trees and
C. 4 1 2 3 plants are cooler at night, there is more
evaporation of water and hence there
D. 4 3 1 2 is less dew formation.
Explanation:
• But when it is cloudy, trees and plants
• Troposphere: temperature falls with do not get cool in the night and hence
height there is less dew formation.
• Tropopause: fall in temperature stops • As the sun rises high in the sky, these
dew drops evaporate into air.
• Stratosphere: contains most of the
ozone
• Ionosphere: reflects radio waves back 18. Consider the following statements with
to the earth reference to Convection Current Theory:
1. Arthur Holmes had discussed the
possibility of convection currents
17. Why are DEWDROPS not formed on a cloudy
operating in the mantle portion.
night?
2. Wherever rising limbs of these
A. Clouds absorb the radiation released
currents meet, trenches are formed on
from the Earth’s surface.
the seafloor and wherever the failing
B. Clouds reflect back the Earth’s limbs meet, oceanic ridges are formed.
radiation
Which of the above statements is/are correct?
C. The Earth’s surface would have low
A. 1 only
temperatures on cloudy nights.
B. 2 only
D. Clouds deflect the blowing wind to
ground level C. Both 1 and 2

Answer: B D. Neither 1 nor 2


Explanation: Answer: A
Option B is correct: Explanation:

• The ground becomes much cooler, • Statement 1 is correct: Convectional


Current Theory was given by Arthur Ridge forms an interconnected chain of
Holmes in the 1930s. It discussed the mountain systems within (not over)
possibility of convection currents the ocean.
operating in the mantle portion These
• Statement 2 is correct: Most volcanism
currents are generated due to
on the planet, including the formation
radioactive elements causing thermal
of volcanoes and volcanic eruptions,
differences in the mantle portion.
occurs along the mid-ocean ridge,
• Statement 2 is incorrect: Wherever where the new oceanic crust is born,
rising limbs of these currents meet, made of lavas and sediments.
oceanic ridges are formed on the
seafloor and wherever the failing
limbs meet, trenches are formed. 20. Who among the following proposed the idea
of Seafloor Spreading?
A. David Henry
19. Consider the following statements with
reference to Mid-Oceanic Ridges: B. Harry Hess

1. It is an interconnected chain of C. Allen J. Scott


mountain systems over the ocean. D. Alfred Wegener
2. It is the zone of intense volcanic Answer: B
activity.
Explanation:
Which of the above statements is/are correct?
Option B is correct:
A. 1 only
• Sea-floor spreading — In the early
B. 2 only 1960s, Princeton geologist Harry Hess
C. Both 1 and 2 proposed the hypothesis of sea-floor
spreading, in which basaltic magma
D. Neither 1 nor 2
from the mantle rises to create new
Answer: B ocean floor at mid-ocean ridges.
Explanation: • On each side of the ridge, sea floor
The mid-ocean ridge is an extensive underwater moves from the ridge towards the
mountain range resulting from the constantly deep-sea trenches, where it is
altering tectonic plates, heat, and magma emitted subducted and recycled back into the
from the earth’s core. This phenomenon has mantle.
occurred since the earth’s evolution, including its • A test of the hypothesis of sea-floor
land and water forms. They are formed due to the spreading was provided by studies of
continuous spread of the oceanic lithosphere, the the Earth’s magnetism.
crust, and the upper mantle at the divergent plate
boundaries due to convection currents that lift the
ocean floor. 21. Which of the following is not an indirect
source of information about the Earth’s Interior?
• Statement 1 is incorrect: Mid- Oceanic
A. Seismic waves Explanation:
B. Meteorites Statement 1 is correct:
C. Gravitation • The gravitational force differs
according to the mass of material.
D. Volcanic eruption
Answer: D • The uneven distribution of mass of
material within the earth influences
Explanation: this value. Such a difference is called a
• Option D is correct: Mining, drilling gravity anomaly.
and volcanic eruption are examples of
• Gravity anomalies give us information
direct sources.
about the distribution of mass in the
Indirect Sources of information about the Earth’s crust of the earth.
Interior
Statement 2 is correct:
• Increase in pressure and temperature
• Meteorites and Earth are born from the
with depth
same nebular cloud. Thus, they are
• Seismic waves likely to have a similar internal
structure.
• Meteorites
• When meteoroids fall to earth, their
• Gravitation
outer layer is burnt during their fall
• Magnetic field due to extreme friction and the inner
core is exposed.

22. Consider the following statements with • The heavy material composition of
reference to the indirect source of about the their cores confirms the similar
Earth’s Interior: composition of the inner core of the
1. Gravity anomalies give information earth.
about the distribution of mass in the
crust of the earth. 23. Which of the following pairs of
2. Meteorites and Earth are born from the discontinuities/ Transition Zones inside the earth
same nebular cloud and thus are likely is/are incorrectly matched?
to have a similar internal structure. 1. Conrad Discontinuity: Transition
Which of the above statements is/are incorrect? zone between Outer mantle and Inner
mantle.
A. 1 only
2. Mohorovicic
B. 2 only
Discontinuity: Transition zone
C. Both 1 and 2 between the Crust and Mantle.
D. Neither 1 nor 2 3. Repetti Discontinuity: Transition
Answer: D zone between upper and lower Crust.
4. Gutenberg Discontinuity: Transition asthenosphere. It is in liquid state.
zone between Mantle and Core. D. The core is sometimes referred to as the
Select the correct answer using the code given Nife layer.
below:
Answer: C
A. 1 and 2 only
Explanation:
B. 3 and 4 only
• Statement 3 is incorrect: The lower
C. 1 and 3 only mantle extends beyond the
D. 2 and 4 only asthenosphere. It is in solid-state.

Answer: C Additional Information:

Explanation: • The lithosphere is the solid, outer part


of the Earth.
• Statement 1 is incorrect: Conrad
Discontinuity: Transition zone • The lithosphere includes the brittle
between upper and lower Crust. upper portion of the mantle and the
crust, the outermost layers of Earth’s
• Statement 3 is incorrect: Repetti
structure.
Discontinuity: Transition zone
between Outer mantle and Inner • It is bounded by the atmosphere above
mantle. and the asthenosphere (another part of
the upper mantle) below.

25. Consider the following statements:


24. Which of the following statements 1. Wadati Benioff zone is a zone of
is incorrect? subduction along which earthquakes
A. Lithosphere includes the brittle upper are very rare.
portion of the mantle and the crust. 2. Usually, the most powerful
B. The upper portion of the mantle is earthquakes along convergent
called the asthenosphere. boundary occur along this zone.

C. The lower mantle extends beyond the Which of the above statements is/are correct?
A. 1 only Explanation:
B. 2 only Basalt, gabbro and dolerite are typical examples of
Basic Rocks.
C. Both 1 and 2
Additional Information:
D. Neither 1 nor 2
• Rocks can be categorized as Acidic,
Answer: B
intermediate or basic, depending on
Explanation: the amount of the elements of silica,
• Statement 1 is incorrect: Wadati SiO2, they contain.
Benioff zone is a zone of subduction • Acidic rocks such as granite,
along which earthquakes are common. microgranite and rhyolite are rich in
• Statement 2 is correct: Wadati Benioff silica and contain the minerals quartz,
zone is a zone of subduction along feldspar and biotite among others.
which earthquakes are common. The • Basic rocks such as gabbro, dolerite
most powerful earthquakes occur and basalt are poor in silica and
along this zone (most powerful contain the minerals olivine, pyroxene,
earthquakes occur along the feldspar and/or quartz among others;
convergent boundary). they are also rich in the metals
Additional information: magnesium and iron and are often
described as “mafic”. The intermediate
• Differential motion along the zone
rocks include diorite, microdiorite and
produces numerous earthquakes, the
andesite.
foci of which may be as deep as about
700 kilometers.
• Wadati–Benioff zones can be produced 27. Consider the following statements with
by slip along the subduction thrust reference to Sedimentary rocks:
fault (Himalayan Region – C-C 1. Ice deposited sedimentary rocks are
convergent boundary) or slip-on faults called tillite.
within the downageing plate (O-O and
2. Wind-deposited sediments are called
C-O convergent boundary).
loess.
3. Sandstone is an example of organically
26. Which of the following is not an example of formed sedimentary rock.
Acidic Rocks?
Which of the above is/are correct?
A. Granite
A. 1 only
B. Quartz
B. 1 and 2 only
C. Feldspar
C. 3 only
D. Dolerite
D. 2 and 3 only
Answer: D
Answer: B
Explanation: B. 2 only
• Statement 1 is correct: unsorted C. 1 and 3 only
sedimentary rock with grains D. 1, 2 and 3
deposited by ice is a tillite. The loose
sediment is called till in its unlithified Answer: D
form Explanation:
• Statement 2 is correct: Loess can be • Statement 1, 2 and 3 are correct:
defined as sediment that has been All are causes of Metamorphism.
entrained, transported, and deposited
by the wind and is dominated by silt- • Orogenic (Mountain Building)
sized (50–2 μm diameter) particles. Movements: Such movements often
Most loess deposits are not composed take place with an interplay of folding,
completely of silt, but also contain warping, and high temperatures.
measurable amounts of sand (> 50 μm) These processes give existing rocks a
and clay (< 2 μm). new appearance.

• Statement 3 is incorrect: Sandstone is • Lava Inflow: The molten magmatic


an example of mechanically formed material inside the earth’s crust brings
sedimentary rock. the surrounding rocks under the
influence of intense temperature
Additional Information: pressure and causes changes in them.
Depending upon the mode of formation,
• Geodynamic Forces: The omnipresent
sedimentary rocks are classified into
geodynamic forces such as plate
• mechanically formed — sandstone, tectonics also play an important role in
conglomerate, limestone, shale, loess. metamorphism.
• organically formed — geyserite, chalk,
limestone, coal.
29. Which of the following statements with
• chemically formed — limestone, halite, reference to Caldera is incorrect?
potash. A. Lake Toba is the most familiar
example.
28. Which of the following are causes of B. A caldera is a large depression formed
Metamorphism? when a volcano erupts and collapses.
1. Orogenic Movements C. Some calderas form a lake as the bowl-
2. Lava Inflow shaped depression fills with water.

3. Geodynamic Forces D. None of the above.

Select the correct answer using the code given Answer: D


below: Explanation:
A. 1 only • Statement A is correct: Due to their
unstable environments, some crater 30. Consider the following pairs of Intrusive
lakes exist only intermittently. Caldera igneous rocks and their forms:
lakes, in contrast, can be quite large 1. Lopoliths: Wavy materials which have
and long-lasting For example, Lake a definite conduit to source beneath.
Toba (Indonesia) formed after its super
2. Phacolith: They are saucer-shaped,
volcanic eruption around 75,000 years
concave to the sky.
ago. It is the largest crater lake in the
world. 3. Sills: These are near horizontal bodies
of the intrusive igneous rocks.
• Statement B is correct: A caldera is a
large depression formed when a Which of the above is/are correct?
volcano erupts and collapses. During a A. 1 only
volcanic eruption, magma present in
B. 3 only
the magma chamber underneath the
volcano is expelled, often forcefully. C. 1 and 2 only

• Statement C is correct: Some calderas D. 2 only


form a lake as the bowl-shaped Answer: B
depression fills with water. A famous
Explanation:
example is Crater Lake, in Oregon.
This caldera formed about 7,000 years • Statement 1 is incorrect: Lopoliths:
ago when a stratovolcano, Mt. Mazama They are saucer-shaped, concave to the
violently erupted. sky.
Additional Information: • Statement 2 is incorrect: Phacolith:
Wavy materials which have a definite
• In some volcanoes, the magma
conduit to source beneath.
chamber below the surface may be
emptied after volcanic eruptions. • Statement 3 is correct: Sills form when
magma intrudes between the rock
• The volcanic material above the
layers, forming a horizontal or gently-
chamber collapses into the empty
dipping sheet of igneous rock.
magma chamber, and the collapsed
surface appears like a large cauldron- Additional Information:
like hollow (tub shaped) called the
caldera.
• When water from rain or melted snow
gets accumulated in the caldera, it
becomes a caldera lake (in general, the
caldera lakes are also called crater
lakes).
31. Consider the following pairs of volcanoes and • Statement 3 is correct: Cinder Cone
their characteristics: Volcanoes are small volcanoes. These
1. Shield Volcanoes: These are volcanoes consist almost entirely of
characterized by low-explosivity. loose, grainy cinders and almost no
lava. They have very steep sides and
2. Caldera: They are characterized by the
usually have a small crater on top.
eruption of cooler and more viscous
lavas than basalt.
3. Composite Volcanoes: These are the 32. Consider the following statements:
most explosive of the earth’s 1. Continental Drift Theory - explains
volcanoes. the movement of Lithospheric plates
4. Cinder Cone Volcanoes: These that include both Continents and
volcanoes consist almost entirely of Oceans.
loose, grainy cinders and almost no 2. According to Plate tectonics -
lava. Convection Currents in the mantle
Which of the above is/are correctly matched? drag crustal plates.

A. 1 and 2 only 3. Continental Drift Theory - helped in


the evolution of Convection Current
B. 3 and 4 only
theory and Seafloor Spreading theory.
C. 1 and 4 only Which of the above is/are correct?
D. 2 and 3 only A. 1 only
Answer: C B. 1 and 3 only
Explanation:
C. 2 only
• Statement 1 is correct: A shield
D. 2 and 3 only
volcano is a type of volcano named for
its low profile, resembling a warrior's Answer: D
shield lying on the ground. It is formed Explanation:
by the eruption of highly fluid lava,
• Statement 1 is incorrect: Continental
which travels farther and forms
Drift Theory explains the movement of
thinner flows than the more viscous
Continents only.
lava erupted from a stratovolcano.
• Statement 2 is incorrect: Caldera:
These are the most explosive of the
earth’s volcanoes.
• Statement 3 is incorrect: Composite
Volcanoes: They are characterized by
the eruption of cooler and more
viscous lavas than basalt.
34. Which of the following statements
is incorrect?
A. Surface Waves are also called long
period waves because of their long
wavelength.
B. Love waves are the fastest surface
wave and move the ground from side-
to-side.
C. Most of the shaking and damage from
an earthquake is due to the Rayleigh
• Statement 2 is correct: Convection wave.
currents drive the movement of
D. None of the above.
Earth's rigid tectonic plates in the
planet's fluid molten mantle. In places Answer: D
where convection currents rise up Explanation:
towards the crust's surface, tectonic
• Option A is correct: Surface Waves are
plates move away from each other in a
also called long period waves because
process known as seafloor spreading
of their long wavelength. They are
• Statement 3 is correct: Continental low–frequency transverse waves
Drift Theory helped in the evolution of (shear waves).
Convection Current theory and
• Option B is correct: Love waves have a
Seafloor Spreading theory.
horizontal motion that moves the
surface from side to side perpendicular
33. Which of the following earthquake waves is to the direction the wave is traveling.
first recorded on the Seismograph? Of the two surface waves, Love waves
move faster.
A. P-waves
• Option C is correct: Rayleigh waves,
B. S-waves
also called ground roll, travel like
C. Rayleigh waves ocean waves over the surface of the
D. Love waves Earth, moving the ground surface up
and down. They cause most of the
Answer: A
shaking at the ground surface during
Explanation: an earthquake
• P waves are the first waves that are
detected by a seismograph. They are
35. Which of the following are true with regard
the fastest seismic waves and can
to Slip Strike earthquakes?
travel through gases, liquids, or solids.
1. These occur when two plates slide
horizontally against one another
causing deformities that occur in plates C. 2 only
distant from fault lines. D. 2 and 3 only
2. They can lead to inter-plate Answer: B
earthquakes and cause a plate to break,
Explanation:
resulting in a new boundary and this,
in turn, can lead to even more quakes. • Statement 1 is correct: Igneous rocks
A. 2 only are called Primary or Mother Rocks
because all other rocks are directly or
B. Both 1 and 2 indirectly formed from them.
C. None of the above.
• Statement 2 is incorrect: If magma
D. 1 only cools slowly at great depths, mineral
Answer: B grains formed in the rocks may be very
large.
Explanation:
o Such rocks are called intrusive rocks
• Statement 1 is correct: strike-slip or plutonic rocks (e.g. Granite).
earthquakes occur along slip strike
o These rocks appear on the surface
faults on which the two blocks slide
past one another. The San Andreas only after being uplifted and
Fault is an example of a right lateral denuded.
fault. • Statement 3 is correct: The flood basalt
province known as the Deccan Traps is
• Statement 2 is correct: They can lead
located on the Deccan Plateau in west-
to inter-plate earthquakes and cause a
central India and is one of the largest
plate to break, resulting in a new
boundary and this, in turn, can lead to volcanic provinces in the world. The
even more quakes. Deccan Plateau consists of more than
2000 m-thick succession of flat-lying
basalt lava flows and covers an area of
36. Consider the following statements: nearly 500 000 km2.
1. Igneous rocks are also known as
primary rocks. 37. Which of the following statements
2. If magma cools slowly at great depths, is incorrect with reference to Metamorphic Rocks
mineral grains formed in the rocks in India?
may be very small. A. The gneisses and schists are commonly
3. The Deccan traps in the Indian found in the Himalayas.
peninsular region are of basaltic origin. B. Quartzite is a hard rock found over
Which of the above is/are correct? Rajasthan.

A. 1 only C. Marble occurs near parts of Narmada


Valley in Madhya Pradesh.
B. 1 and 3 only
D. None of the above. Answer: D
Answer: D Explanation:
Explanation: • All statements are correct.
• Option, A, B and C are correct: Additional Information
Metamorphic Rocks in India • Evidence in Support of the Continental
Drift:
• The gneisses and schists are
commonly found in the Himalayas, • The Matching of Continents (Jig-Saw-
Assam, West Bengal, Bihar, Orissa, Fit): The shorelines of Africa and South
Madhya Pradesh and Rajasthan. America facing each other have a
• Quartzite is a hard rock found over remarkable and unmistakable match.
Rajasthan, Bihar, Madhya Pradesh, • Rocks of Same Age Across the
Tamil Nadu and areas surrounding Oceans: The belt of ancient rocks of
Delhi. 2,000 million years from Brazil coast
• Marble occurs near Alwar, Ajmer, matches with those from western
Jaipur, Jodhpur in Rajasthan and Africa.
parts of Narmada Valley in Madhya • Tillite: It is the sedimentary rock
Pradesh. formed out of deposits of glaciers.
• Slate, which is used as a roofing • The Gondawana system of sediments
material and for writing in schools, is from India is known to have its counter
found over Rewari (Haryana), Kangra parts in six different landmasses of the
(Himachal Pradesh) and parts of Bihar. Southern Hemisphere.
• Graphite is found in Orissa and • Overall resemblance of the
Andhra Pradesh. Gondawana type sediments clearly
demonstrates that these landmasses
had remarkably similar histories.
38. Which of the following are evidence to
continental drift theory? • The occurrence of rich placer deposits
of gold in the Ghana coast and the
1. Tillite deposits.
absolute absence of source rock in the
2. Placer Deposits. region is an amazing fact. The gold
3. Rocks of Same Age Across the Oceans. bearing veins are in Brazil and it is
obvious that the gold deposits of the
Select the correct code:
Ghana are derived from the Brazil
A. 1 only plateau when the two continents lay
B. 3 only side by side.

C. 1 and 2 only
D. All of the above. 39. Consider the following statements regarding
Ring of Fire:
1. The Ring of Fire is a direct result of geothermal reserves. More than 200
plate tectonics/ movement and volcanoes are located along Sumatra,
collisions of lithospheric plates. Java, Bali and the islands of eastern
part of Indonesia, which is known as
2. About 90%of the world's earthquakes
The Ring of Fire.
occur along the Ring of Fire.
Source: GC Leong geography
3. Indonesia is the country of the Ring of
Fire volcanic belt and holds about 40%
of the world's geothermal reserves. 40. Which of the statements mentioned below
Which of the above statements is/are correct? is/are true about Mohorovicic discontinuity?
A. 1 only 1. It is the boundary between Mantle and
the Core of the earth.
B. 3 only
2. It is uniform across the surface of the
C. 1 and 3 only
continents and below the seafloor.
D. All of the above.
Choose the correct answer using the codes from
Answer: D below:
Explanation: • 1 only
The Ring of Fire (also known as the Rim of Fire or • 2 only
the Circum-Pacific belt) is a major area in the basin
• 1 and 2 only
of the Pacific Ocean where many earthquakes and
volcanic eruptions occur. • Neither 1 nor 2

• Statement 1 is correct: The Ring of Fire Answer: D


is a direct result of plate tectonics: the Explanation:
movement and collisions of
• Statement 1 is incorrect : Mohorovicic
lithospheric plates, especially
discontinuity is boundary between
subduction in the northern portion. It
Earth’s crust and Mantle.
is associated with a nearly continuous
series of oceanic trenches, volcanic • Statement 2 is incorrect:It is not
arcs, and volcanic belts and plate uniform and it is depth of 30-50 km
movements. below the continents and 5-10 km
below sea level in the ocean.
• Statement 2 is correct: It has more
than 75% of the world's active and Additional Info:
dormant volcanoes. About 90% of the • In geology the word "discontinuity" is
world's earthquakes and about 81% of used for a surface at which seismic
the world's largest earthquakes occur waves change velocity.
along the Ring of Fire.
• Mohorovicic Discontinuity occurs at
• Statement 3 is correct: Indonesia is the an average depth of about 8 kilometres
country of Ring of Fire volcanic belt beneath the ocean basins and 32
and holding about 40% of the world's
kilometres beneath continental flying time than westbound flights
surfaces. because of help from the fast-moving
air. Jet streams can contain wind shear,
a violent and sudden change in wind
direction and speed, which is a major
threat in air travel. Wind shear has
caused airliners to suddenly lose
altitude, putting them in danger of
crashing. In 1988, the FAA decided that
all commercial aircraft must have
wind-shear warning systems, but it
wasn’t until 1996 that all airlines had
them on-board.
• Statement 2 is correct: The monsoon of
South Asia is largely affected and
controlled by jet streams
41. Consider the following statements with
reference to Jet Streams: • Statement 3 is correct: The close
relationship between the intensity of
1. With the help of Jet Stream the
Mid-latitude cyclones and jet streams.
westbound flights usually take less
The cyclones become very strong and
flying time than eastbound flights
stormy when the upper air
2. The monsoon of South Asia is largely tropospheric jet streams are positioned
affected and controlled by jet streams above temperate cyclones
3. The position of upper air tropospheric
jet streams above temperate cyclones
42. Which of the following statements is
makes the cyclones strong and stormy.
incorrect with reference to Jet Streams?
Which of the above statements is/are
A. Polar Front Jet Streams move in
correct?
easterly direction and are regular
A. 1 only
B. Subtropical Westerly jet Streams are
B. 2 and 3 only also known as stratospheric subpolar
C. 3 only jet streams

D. 1 and 2 only C. Tropical Easterly jet streams play an


important role in Indian Monsoon
Answer: B
D. Polar Night Jet Streams develop in
Explanation:
winter season due to steep
• Statement 1 is incorrect: Jet streams temperature gradient in the
play a major role in air travel. stratosphere around the poles
Eastbound flights usually take less
Answer: A
Option A is incorrect: Polar Front Jet Streams move
in easterly direction and are irregular.
Option B is correct: Subtropical Westerly jet
Streams are also known as stratospheric subpolar
jet streams.
Option C is correct: Tropical Easterly jet streams
play an important role in Indian Monsoon.
Option D is correct: Polar Night Jet Streams
develop in winter season due to steep temperature
gradient in the stratosphere around the poles.
Additional information: Subtropical Westerly jet Streams

• Types of jet Streams • Formed above 30-35 latitude

• Polar front jet streams • Move-in upper troposphere to the


north of the subtropical surface high-
• Subtropical Westerly Jet streams
pressure belt
• Tropical easterly Jet streams
• Also known as stratospheric subpolar
• Polar night Jet Streams jet streams.
• Local Jet Streams Tropical Easterly jet streams
Permanent jet streams – subtropical jets at • Develop in upper troposphere above
lower latitudes and polar front jets at mid surface easterly trade winds over India
latitudes. and Africa during the summer season
Temporary jet streams – Tropical Easterly Jet due to intense heating of Tibetan
or African Easterly Jet, and Somali Jet (south- plateau and play an important role in
westerly). Indian Monsoon.

Polar Front Jet Streams Polar Night Jet Streams

• Formed above the convergence zone • Develop in winter season due to steep
(40-60 degree) of surface polar cold air temperature gradient in the
mass and tropical warm air mass. stratosphere around the poles.

• These move in easterly direction but Local Jet Streams


are irregular • Formed locally due to local thermal
and dynamic conditions and have
limited local importance.

43. Due to which of the following reasons


the unlike air masses on meeting do not
merge readily?
1. Effect of the converging atmospheric Statement 1 is correct: The Earth’s major air masses
circulation originate in polar or subtropical latitudes.
2. Relatively low diffusion coefficient Statement 2 is correct: Areas with high pressure but
Low thermal conductivity little pressure difference are ideal source regions.
Select the correct answer using the codes Statement 3 is correct: There are no major source
given below: regions in the mid-latitudes as these regions are
A. 1 and 2 only dominated by cyclonic and other disturbances.

B. 2 and 3 only
C. 1 and 3 only 45. Which of the following is/are favorable
conditions for Tropical Cyclone Formation?
D. 1, 2 and 3
1. Large sea surface with a temperature
Answer: D higher than 27° C
When unlike air masses meet, the two air masses 2. Presence of the Coriolis force enough
don’t merge readily due to the effect of the to create a cyclonic vortex
converging atmospheric circulation, relatively low
diffusion coefficient, and low thermal 3. Small variations in the vertical wind
conductivity. speed
4. A pre-existing weak low-pressure area
or low-level-cyclonic circulation
44. Consider the following statements with
reference to Air Masses: 5. Upper divergence above the sea level
system
1. The Earth’s major air masses originate
in polar or subtropical latitudes. Select the correct code:

2. Areas with high pressure but little A. 1, 2 and 3 only


pressure difference are ideal source B. 4 and 5 only
regions.
C. 1, 4 and 5 only
3. There are no major source regions in
D. 1,2,3,4 and 5
the mid-latitudes as these regions are
dominated by cyclonic and other Answer: D
disturbances. • Conditions Favorable for Tropical
Which of the above statements is/are Cyclone Formation
correct? • Large sea surface with a temperature
A. 1 only higher than 27° C,

B. 2 and 3 only • Presence of the Coriolis force enough


to create a cyclonic vortex,
C. 3 only
• Small variations in the vertical wind
D. 1, 2 and 3
speed,
Answer: D
• A pre-existing weak low-pressure area 3. Tropical cyclones follow a parabolic
or low-level-cyclonic circulation, path
• Upper divergence above the sea level Which of the above statements is/are
system correct?
A. 1 and 2 only
46. Consider the following statements: B. 2 and 3 only
1. Tornadoes as well as cyclones both C. 1 and 3 only
occur in India. D. 1, 2 and 3
2. Tornadoes of weak strength occur in Answer: D
the north-western and north-eastern
• Statement 1 is correct: Tropical
region of India
cyclones have symmetrical elliptical
Which of the above statements is/are shapes (2:3 ratio of length and breadth)
correct? with steep pressure gradients. They
A. 1 only have a compact size—80 km near
center, which may develop up to 300
B. 2 only
km to 1500 km.
C. Both 1 and 2
• Statement 2 is correct: Wind Velocity
D. Neither 1 nor 2 and StrengthWind velocity, in a
Answer: C tropical cyclone, is more in poleward
margins than at center and is moreover
Statement 1 is correct: Tornadoes, as well as
oceans than over landmasses, which
cyclones both, occur in India. However, unlike
are scattered with physical barriers.
cyclones, the frequency of tornado outbreaks is
The wind velocity may range from nil
very low.
to 1200 km per hour.
Statement 2 is correct: Cyclones originate in the
• Statement 3 is correct: Path of Tropical
Bay of Bengal region as well as in the Arabian Sea
CyclonesThese cyclones start with a
region whereas Tornadoes of weak strength occur
westward movement but turn
in the north-western and north-eastern region of
northwards around 20° latitude. They
the country causing significant damage to man and
turn further north-eastwards around
material.
25° latitude, and then eastwards
around 30° latitude. They then lose
47. Consider the following statements with energy and subside.
reference to Tropical Cyclones: Tropical cyclones follow a parabolic
path, their axis being parallel to the
1. Tropical cyclones have symmetrical
isobars.
elliptical shapes
• Coriolis force or earth’s rotation,
2. Wind velocity, in a tropical cyclone, is
easterly and westerly winds influence
more in poleward margins than at
the path of a tropical cyclone.
centre
• Tropical cyclones die at 30° latitude 49. Consider the following statements with
because of cool ocean waters and reference to the Heat Budget of the Earth:
increasing wind shear due to 1. The surplus heat energy from the tropics is
westerlies. redistributed towards the pole as tropics do
not get progressively heated up and high
latitudes get permanently frozen.
48. Consider the following points
regarding incoming and outgoing radiation from 2. Roughly 55 percent of insolation is
the earth’s surface: reflected back to space even before
1. Incoming waves are shortwave whereas reaching the earth’s surface.
outgoing waves are longwave radiation. Which of the statements given above is/are
correct?
2. Shortwave radiation contains a lot of
energy whereas a long wave contains less A. 1 only
energy than shortwave radiation. B. 2 only
Which of the following points is correct? C. 1 and 2 only
A. 1 and 2 D. None of these.
B. 1 only Answer: A
C. 2 only Explanation:
D. Neither 1 nor 2 Statement 1 is correct: The surplus heat energy
Answer: A from the tropics is redistributed pole wards and as
Explanation: a result the tropics do not get progressively heated
up due to the accumulation of excess heat or the
Statement 1 is correct: Incoming waves are high latitudes get permanently frozen due to
shortwave whereas outgoing waves are longwave excess deficit.
radiation.
Statement 2 is incorrect: Roughly 35 units are
Statement 2 is correct: Shortwave radiation reflected back to space even before reaching the
(visible light) contains a lot of energy; longwave earth’s surface.
radiation (infrared light) contains less energy than
shortwave radiation. The sun emits shortwave Additional Information
radiation because it is extremely hot and has a lot ● Heat Budget of the Earth is the balance
of energy to give off. Once in the Earth’s between incoming solar insolation and
atmosphere, clouds and the surface absorb the outgoing terrestrial radiation which
solar energy. The ground heats up and re-emits maintains the average annual temperature
energy as long wave radiation in the form of of the earth at 15-degree Celsius.
infrared rays. Earth emits long wave radiation ● Solar energy received by earth’s surface is
because Earth is cooler than the sun and has less called solar insolation.
energy available to give off.
50. Consider the following points 1. It is the reversal of normal behavior in
regarding isotherms: temperature in the stratosphere.
1. It is a line connecting points having the 2. In this a layer of cool air at the surface is
same temperature difference at a given overlain by a layer of warmer air.
time or average of the time period given.
Which of the following points is correct?
2. Isotherms run along the longitudes, but
A. 1 and 2
they are not parallel to the longitudes.
B. 2 only
Which of the following points is correct?
C. None of the above.
A. 1 and 2
D. All of the above.
B. 2 only
Answer: - B
C. None of the above.
Explanation:
D. All of the above.
Statement 1 is incorrect: Temperature
Answer: - C Inversion, a reversal of the normal behaviour of
Explanation: temperature in the troposphere (the region of the
Statement 1 is incorrect: Isotherms - A line on a atmosphere nearest the Earth’s surface).
map connecting points having the same Statement 2 is correct: In this, a layer of cool air at
temperature at a given time or on average over a the surface is overlain by a layer of warmer air.
given period. (Under normal conditions air temperature usually
Statement 2 is incorrect: Isotherms run along the decreases with height.)
latitudes, but they are not parallel to the latitudes.
Generally there and straight but bent at the 52. With reference to Coriolis force, which of the
junction of continents and oceans. They take following is/are true?
sudden bends at land-water edges because of land-
1. In the southern hemisphere it deflects the
water contact.
wind to the left.
Additional information:
2. The key to the Coriolis effect lies in Earth’s
• Characteristics – Closely spaced rotation
isotherms indicate Rapid rate of
3. The development of weather patterns, such
change of temperature and vice versa.
as cyclones and trade winds, are examples
Irregular in the Northern hemisphere
of the impact of the Coriolis effect.
because of the large extent of
continents, but there are more regular Select the correct answer using the codes given
in the southern hemisphere due to over below:
dominance of oceans A. Only 1
B. Only 1 and 2
51. Consider the following points C. Only 2 and 3
regarding Temperature inversion:
D. 1, 2 and 3
Answer: - D
Explanation: 53. Consider the following factors which affect
the wind velocity:
Statement 1 is correct: It deflects the wind to the
right direction in the northern hemisphere and to 1. Pressure gradient force
the left in the southern hemisphere. The deflection 2. Frictional force
is more when the wind velocity is high.
3. Coriolis force
Statement 2 is correct:
4. Gravitational pull of the earth
● The key to the Coriolis effect lies in Earth’s
rotation. Specifically, Earth rotates faster at Which of the following points is correct?
the Equator than it does at the poles. A. 1, 2 and 3
● Earth is wider at the Equator, so to make a B. 2 and 3
rotation in one 24-hour period, equatorial
C. 1, 3 and 4
regions race nearly 1,600 kilometers (1,000
miles) per hour. D. All of the above.

● Near the poles, Earth rotates at a sluggish Answer: - A


0.00008 kilometers (0.00005 miles) per hour. Explanation:
Statement 3 is correct: While the first 3 are the factors of wind velocity but
● The development of weather patterns, such gravitational pull of earth has nothing to do with
as cyclones and trade winds, are examples the wind velocity.
of the impact of the Coriolis effect.
Cyclones are low-pressure systems that 54. Which of the following is paired incorrectly?
suck air into their center, or “eye.”
A. Chinook – USA and Canada along
● In the Northern Hemisphere, fluids from rockies
high-pressure systems pass low-pressure
systems to their right. B. Foehn– Alps mountains in Switzerland

● As air masses are pulled into cyclones from C. Harmattan – Dry winds in the the parts
all directions, they are deflected, and the of Atacama desert
storm system—a hurricane—seems to D. Sirocco - Mediterranean wind that
rotate counter-clockwise. comes from the Sahara
● In the Southern Hemisphere, currents are Answer: - C
deflected to the left. As a result, storm Explanation:
systems seem to rotate clockwise.
Option c is correct: HARMATTAN The warm and
● The pressure gradient force is dry winds blowing from north-east and east to
perpendicular to an isobar. The higher the west in the parts of Sahara Desert are called
pressure gradient force, the more is the Harmattan. These winds become extremely dry
velocity of the wind and the larger is the because of their journey over the Sahara desert.
deflection in the direction of wind.
This is why Harmattan is known as "Doctor' in the 2. Maritime air masses contain high humidity
Guinea coastal area of western Africa. and produce a large amount of
precipitation whereas continental air
masses are dry and produce less amount of
55. Consider the following statements: (PYQ) precipitation.
1. Jet streams occur in the Northern Which of the following points is correct?
Hemisphere only.
A. 1 and 2
2. Only some cyclones develop an eye.
B. 2 only
3. The temperature inside the eye of a cyclone
C. 1 only
is nearly 10°C less than that of the
surroundings. D. None of the above.
Which of the statements given above is/are Answer: - A
correct? Explanation:
A. 1 only Statement 1 is correct:
B. 2 and 3 only ● When the air remains over a homogenous
C. 2 only area for a sufficiently long time, it acquires
the characteristics of the area.
D. 1 and 3 only
● The homogenous regions can be the vast
Answer: - C
ocean surface or vast plains.
Explanation:
Statement 2 is correct:
Statement 1 is incorrect: Jet streams are high
● Maritime air masses contain high humidity
altitude westerly wind system blows at a height of
and produce a large amount of
6 to 14 km, with very high speed up to 450 km/hr
precipitation. Continental air masses are
in wavy form at both hemispheres.
dry and produce less amount of
Statement 2 is correct: Centre of the cyclone is precipitation.
called the Eye. The eye is a Calm region with no
rainfall and experiences highest temperature and
lowest pressure within the cyclonic system. 57. Consider the following statements
about temperate cyclones.
Statement 3 is incorrect: eye region is warmer and
not colder, hence statement is wrong. 1. They are low-pressure systems with
associated cold fronts, warm fronts, and
occluded fronts.
56. Consider the following statements with
2. These are more frequent in the northern
reference to air mass:
hemisphere, especially on the European
1. When the air remains over a homogenous mainland.
area for a sufficiently long time and it
Which of the above is/are correct?
acquires the characteristics of the area.
A. 1 only
B. 2 only ● The warm air climbs over the cold air and a
series of clouds appear over the sky ahead
C. Both 1 and 2
of the warm front and cause rainfall.
D. Neither 1 nor 2
● The cold front approaches the warm air
Answer: - A from behind and pushes the warm air up.
Explanation: ● As an outcome, cumulus clouds develop
Statement 1 is correct: along the cold front.
● The Extra-Tropical Cyclones are storm ● The cold front moves faster than the warm
systems emerging in the mid and high front, eventually surpassing the warm
latitudes, away from the tropics. front.
● They are low-pressure systems with ● The warm air is entirely lifted and the front
associated cold fronts, warm fronts, and is occluded and the cyclone dissipates.
occluded fronts.
● They can originate over the land and sea
Statement 2 is incorrect: They are more frequent in and cover a larger area.
Southern Hemisphere.
Additional Information:
58. Which of the following statements
Characteristics of Temperate Cyclone (Extra- is incorrect with reference to the difference
Tropical Cyclones) between Fog and Mist?
● These cyclones are formed along the polar A. The difference between the mist and
front. fog is that fog contains more moisture
● In the beginning, the front is stationary. than mist.

● Extra-tropical cyclones are also known as B. Fogs are drier than mist and they are
mid-latitude storms or baroclinic storms. prevalent where warm currents of air
come in contact with cold currents.
● In the Northern hemisphere, cold air blows
from the north of the front and warm air C. In mist visibility is more than one
blows from the south. kilometer but less than two kilometers.

● When the pressure descends along the D. In fog visibility is less than one
front, the cold air moves towards the south, kilometer.
and the warm air moves northwards Answer: - A
setting in motion an anticlockwise cyclonic Explanation:
circulation.
Statement 1 is incorrect: The difference between
● The cyclonic circulation results in a well- mist and fog is that mist contains more moisture
built extratropical cyclone, with a cold front than fog.
and a warm front.
Additional Information
● There are pockets of warm air compressed
● In mist each nucleus contains a thicker
between the forward and the rear cold air.
layer of moisture.
● Mists are frequent over mountains as the 2. The convective transfer of energy is
rising warm air up the slopes meets a cold confined only to the Mesosphere.
surface. Which of the above is/are correct?
● Mist is also formed by water droplets, but A. 1 only
with less merging or coalescing. This
means mist is less dense and quicker to B. 2 only
dissipate. C. Both 1 and 2
D. Neither 1 nor 2
59. Consider the following statement with Answer: - A
reference to Water Vapour:
Explanation:
1. It absorbs not only the long-wave terrestrial
Statement 1 is correct: The air in contact with the
radiation but also a part of the incoming
earth upsurges vertically on heating in the form of
solar radiation.
currents and transfers the heat of the atmosphere.
2. On condensation, it releases latent heat of This vertical heating of the atmosphere is known as
condensation —the ultimate driving force convection.
behind all storms.
Statement 2 is incorrect: The convective transfer of
Which of the above is/are correct? energy is confined only to the Troposphere.
A. 1 only
B. 2 only 61. A layer in the Earth’s atmosphere called
C. Both 1 and 2 Ionosphere facilitates radio communication.
Why? ( UPSC - 2011)
D. Neither 1 nor 2
1. The presence of ozone causes the reflection
Answer: - C of radio waves to Earth.
Explanation: 2. Radio waves have a very long wavelength.
Statement 1 is correct: It absorbs not only the long- Which of the statements given above is/are
wave terrestrial radiation (infrared or heat emitted correct?
by the earth during nights) but also a part of the
A. 1 only
incoming solar radiation.
B. 2 only
Statement 2 is correct: Water vapor is the source of
precipitation and clouds. On condensation, it C. Both 1 and 2
releases latent heat of condensation —the ultimate D. Neither 1 nor 2
driving force behind all storms.
Answer: - D
Explanation:
60. Consider the following statements:
Statement 1 is incorrect: Due to the ability of
1. The process of vertical heating of the ionized atmospheric gases to refract high
atmosphere is known as convection. frequency (HF, or shortwave) radio waves, the
ionosphere can reflect radio waves directed into C. 1, 2 and 3
the sky back toward the Earth. D. 1 and 3 only
Statement 2 is incorrect: Radio waves have low Answer: - C
wavelength.
Explanation:
Statement 1 is correct: The lapse rate is the rate at
62. Which of the following statements which an atmospheric variable, normally
is incorrect with reference to Spatial Distribution temperature in Earth's atmosphere, falls with
at the Earth’s Surface? altitude. For every 165 metre rise in altitude, the
A. Equator receives comparatively more temperature decreases by 1 degree Celsius. This is
insolation than the tropics. called the normal lapse rate.
B. Maximum insolation is received over Statement 2 is correct: Inversion is usually of short
the subtropical desert. duration but quite common nonetheless. A long
C. At the same latitude, insolation is more winter night with clear skies and still air is an ideal
over the continent than over the situation for inversion. As clouds trap heat it will
oceans. not favour temperature inversion.

D. None of the above. Statement 3 is correct: Over polar areas,


temperature inversion is normal throughout the
Answer: - A year. This happens due to lack of sunlight in most
Explanation: of the seasons which leads to loss of heat from the
Statement 1 is incorrect: the Equator receives surface. Even during the seasons when it is
comparatively less insolation than the tropics. exposed to sunlight, it loses maximum amount of
surface heat due to high albedo. This leads to a heat
deficit in the area leading to temperature inversion.
63. Consider the following statements:
1. Normally temperature decreases with an 64. Consider the following statements: (PYQ
increase in elevation, called a normal Lapse 2008)
rate.
1. The albedo of an object determines its
2. A long winter night with clear skies and visual brightness when viewed with
still air can have Inversion of temperature reflected light.
i.e. temperature increases with an increase
2. The albedo of Mercury is much greater
in height.
than the albedo of the Earth.
3. Over polar areas temperature inversion is
Which of the statement(s) given above is/are
normal throughout the year.
correct?
Select the correct answer using the code given
A. 1 only
below:
B. 2 only
A. 1 only
C. Both 1 and 2
B. 1 and 2 only
D. Neither 1 nor 2 Statement 2 is correct: Air Drainage protects
Answer: - A plants from frost damages.

Explanation:
Statement 1 is correct: 66. The transport of heat from the equator
towards the poles by the atmosphere is known as:
● Albedo is the fraction of the incident
sunlight that is reflected from the A. Meridional transport of heat
surface of an object. B. Seridional transport of heat
● When an object reflects most of the light C. Longitudinal heat balance
that hits it and looks bright then it has a
D. Heat balance transport system
high albedo. ex: Snow and ice have high
albedo. Answer: - A

Statement 2 is incorrect: Albedo of mercury and Explanation:


the earth is 0.142 and 0.367 respectively. Meridional Transport of Heat
To balance the energy-rich and efficient areas it is
65. Consider the following statements with clear that there must be 2-way transfer heat from
reference to Air Drainage: • The Earth surface to the atmosphere
1. The temperature inversion takes place in • The equator to the poles - This
hills and mountains due to air drainage. transport of heat from the equator
2. Air Drainage protects plants from frost towards the poles by the atmosphere is
damages. known as meridional transport of heat.
The meridional transport of heat
Which of the above is/are correct?
energy in the form of sensible heat is
A. 1 only accomplished by the atmospheric
B. 2 only circulation and ocean currents.

C. Both 1 and 2
D. Neither 1 nor 2 67. In the middle latitudes, the circulation is that
of sinking cold air that comes from the poles and
Answer: - C the rising warm air that blows from the
Explanation: subtropical high. At the surface these winds are
called westerlies and the cell is known as the?
Statement 1 is correct: The inversion takes place in
hills and mountains due to air drainage. Cold air at A. Ferrel cell
the hills and mountains, produced during night, B. Polar cell
flows under the influence of gravity. Being heavy
C. Hadley cell
and dense, the cold air acts almost like water and
moves down the slope to pile up deeply in pockets D. Walker cell
and valley bottoms with warm air above. This is Answer: - A
called air drainage.
Explanation:
● The Ferrel cell is the average motion of air ● bora, originally defined as a very strong
in the mid-latitudes.occurs at higher cold wind that blows from the northeast
latitudes (between 30 degrees and 60 onto the Adriatic region of Italy, Slovenia,
degrees N and 30 degrees and 60 degrees S. and Croatia.
● The mistral is a strong, cold, northwesterly
wind that blows from southern France into
the Gulf of Lion in the northern
Mediterranean.

69. What causes wind to deflect towards the left


in the Southern hemisphere? (PYQ 2010)
A. Temperature
B. Magnetic field
C. Rotation of the Earth
● In the Ferrel cell, air flows poleward and
D. Pressure
eastward near the surface and equatorward
and westward at higher altitudes; this Answer: - C
movement is the reverse of the airflow in Explanation: Because the Earth rotates on its axis,
the Hadley cell. The Ferrel Cell plays a circulating air is deflected toward the right in the
major part in the poleward energy (mainly Northern Hemisphere and toward the left in the
heat) transport. Southern Hemisphere. This deflection is called the
Coriolis effect.

68. Which of the following is not a Cold wind?


A. Pampero 70. When two different air masses meet, the
boundary zone between them is called a front.
B. Bora
Choose the Incorrect statements in context with
C. Mistral the same:
D. Zonda 1. When the cold air moves towards the warm
air mass, its contact zone is called the cold
Answer: - D
front.
Zonda is a warm wind.
2. If the warm air mass moves towards the
Additional Information: cold air mass, the contact zone is a warm
● The pampero is a burst of cold polar air front.
from the west, southwest, or south on the 3. The fronts occur in upper latitudes and are
pampas in the south of Brazil, Argentina, characterized by steep gradients in
Uruguay, Paraguay and Bolivia temperature and pressure.
4. They bring abrupt changes in temperature
and cause the air to rise to form clouds and 71. Which of the following statements is incorrect
cause precipitation. with reference to the Cold Front?
A. 1 only 1. The weather along such a front depends on
B. 2 only a narrow band of cloudiness and
precipitation.
C. 3 only
2. Such fronts cause moderate to gentle
D. 4 only
precipitation over a large area, over several
Answer: - C hours.
Statement c is incorrect: The fronts occur in 3. Cirrostratus clouds ahead of the cold front
middle latitudes and are characterized by steep create a halo around the sun and moon.
gradients in temperature and pressure.
Select the correct answer using the code given
• Fronts: When two different air masses below:
meet, the boundary zone between
A. 1 only
them is called a front.
B. 2 and 3 only
• When two different air masses meet,
the boundary zone between them is C. 1 and 3 only
called a front. D. 2 only
• The process of formation of the fronts Answer: - B
is known as frontogenesis.
Explanation:
• There are four types of fronts: (a) Cold;
Statement 1 is correct: A cold front commonly
(b) Warm; (c) Stationary; (d) Occluded
brings a narrow band of precipitation that follows
.
along the leading edge of the cold front. These
• When the front remains stationary, it is bands of precipitation are often very strong, and
called a stationary front. can bring severe thunderstorms, hailstorms, snow
• When the cold air moves towards the squalls, and/or tornadoes.
warm air mass, its contact zone is Statements 2 and 3 are incorrect: These statements
called the cold front, whereas if the talk about a warm front. Weather along a warm
warm air mass moves towards the cold front:
air mass, the contact zone is a warm
• As the warm air moves up the slope, it
front.
condenses and causes precipitation
• If an air mass is fully lifted above the but, unlike a cold front, the
land surface, it is called the occluded temperature and wind direction
front. changes are gradual.
• They bring abrupt changes in • Such fronts cause moderate to gentle
temperature and cause the air to rise to precipitation over a large area, over
form clouds and cause precipitation. several hours.
• The passage of the warm front is rainfall and experiences highest temperature and
marked by a rise in temperature, lowest pressure within the cyclonic system.
pressure, and change in weather. Statement 3 is incorrect: eye region is warmer and
Clouds along a warm front not colder, hence statement is wrong.
• With the approach, the hierarchy of
clouds is—-cirrus, stratus and nimbus. 73. Which of the following Regional names for
[No cumulonimbus clouds as the Tropical Cyclones is incorrectly matched?
gradient is gentle]
1. Indian Ocean: cyclone
• Cirrostratus clouds ahead of the warm
front create a halo around the sun and 2. Atlantic: Hurricane
moon. 3. Western Australia: Typhoons
Select the correct code:
72. Consider the following statements: (PYQ A. 1 only
2020)
B. 1 and 2 only
1. Jet streams occur in the Northern
C. 3 only
Hemisphere only.
D. 2 and 3 only
2. Only some cyclones develop an eye.
Answer: - C
3. The temperature inside the eye of a cyclone
is nearly 10°C less than that of the Explanation:
surroundings.
What they are
Regions
Which of the statements given above is/are called
correct?
Indian Ocean Cyclones
A. 1 only
B. 2 and 3 only Atlantic Hurricanes

C. 2 only Western Pacific and


Typhoons
D. 1 and 3 only South China Sea

Answer: - C Western Australia Willy-willies


Explanation:
Statement 1 is incorrect: Jet streams are high
altitude westerly wind system blows at a height of 74. Which of the following characteristics of
Temperate Cyclones is incorrect?
6 to 14 km, with very high speed up to 450 km/hr
in wavy form at both hemispheres. A. The temperate cyclones are
asymmetrical and shaped like an
Statement 2 is correct: Centre of the cyclone is
inverted ‘V’.
called the Eye. The eye is a Calm region with no
B. They stretch over 500 to 600 km.
C. The wind strength is more over Europe ● Dominated by Maritime Tropical air
compared to North America. masses.
D. None of the above. ● Temperature is uniform throughout the
Answer: - C year.

Explanation: ● The mean monthly temperatures are


always around 27° C with very little
Statement 3 is incorrect: The wind strength is more
variation.
in eastern and southern portions, moreover North
America compared to Europe. ● There is no winter. [Typical to Equatorial
Rainforest Climate.
Additional Information:
● Cloudiness and heavy precipitation
● The temperate cyclones are asymmetrical
moderate the daily temperature.
and inverted 'V-shaped. They cover a
distance of 500 to 600 kilometres. ● Regular land and sea breezes assist in
maintaining a truly equable climate.
● They may cover a distance of 2500
kilometres throughout North America ● The diurnal range of temperature is small,
(Polar Vortex). They range in height from 8 and so is the annual range.
to 11 kilometres. Precipitation
● Precipitation is heavy and well distributed
75. Which of the following is not a characteristic throughout the year.
of Equatorial Climate? ● Annual average is always above 150 cm. In
A. There is no winter. some regions the annual average may be as
high as 250 – 300 cm.
B. The diurnal range of temperature is
small, but the annual range of ● There is no month without rain (distinct
temperature is high. dry season is absent). The monthly average
is above 6 cm most of the time.
C. The double rainfall peaks coinciding
with the equinoxes are a characteristic ● There are two periods of maximum rainfall,
feature of equatorial climates not April and October. [shortly after the
found in any other type of climate. equinox]. Least rainfall occurs in June and
December [solstice].
D. None of the above.
● The double rainfall peaks coinciding with
Answer: B
the equinoxes are a characteristic feature of
Explanation: equatorial climates not found in any other
Statement 2 is incorrect: The diurnal range of type of climate.
temperature is small, and so is the annual range. ● There is much evaporation and convection
Additional Info: air currents are set up, followed by heavy
thunderstorms in the afternoons.
Equatorial Climate
This climate is characterized by cold, dry winters
and warm, wet summers. Winter ● The other region is the eastern coastlands of
temperatures are below freezing-point and Asia, including
snowfall is quite natural. Summers are as
eastern Siberia, North China, Manchuria,
warm as the tropics. Rainfall occurs
Korea and northern Japan. It may be
throughout the year with summer maxima.
referred to as the Asiatic region. In the
Dry Westerlies that blow from continental
southern hemisphere, this climatic type is
interiors dominate winters.
absent because only a small section of the
southern continents extends south of the
76. The above statements are with reference to latitude of 40° S.
which of the following Climate?
A. Laurentian Climate 77. Which of the following statements best
B. China Type Climate describes ‘Doldrums’?

C. British Type Climate A. The northeast and southeast trade


winds meet there; this meeting causes
D. Mediterranean Climate
air uplift and often produces clusters of
Answer: A convective thunderstorms.
Explanation: B. The specific position of doldrums is
Laurentian Climate is characterized by cold, dry fixed and does not vary seasonally.
winters and warm, wet summers. Winter C. Tropical region in Indian ocean where
temperatures are below freezing-point and cyclones often originate.
snowfall is quite natural. Summers are as warm as
D. Frigid zone of the earth where there is
the tropics. Rainfall occurs throughout the year
little atmospheric circulation.
with summer maxima. Dry Westerlies that blow
from continental interiors dominate winters. Answer: A

Additional Info: Explanation:

● The Cool Temperate Eastern Margin Statement A is correct: The northeast and
(Laurentian) Climate is an intermediate southeast trade winds meet there; this meeting
type of climate between the British and the causes air uplift and often produces clusters of
Siberian type of climate. convective thunderstorms.

● It has features of both the maritime and the Additional Information


continental climates. ● Doldrums, also called equatorial calms,
● The Laurentian type of climate is found equatorial regions of light ocean currents
only in two regions. One is north-eastern and winds within the intertropical
North America, including eastern Canada, convergence zone (ITCZ), a belt of
north-east U.S.A., (i.e. Maritime Provinces converging winds and rising air encircling
and the New England states), and Earth near the Equator.
Newfoundland. This may be referred to as ● They occur along the Equator in the Indian
the North American region. and western Pacific oceans and slightly
north of the Equator off the African and Which of the statements given above
Central American west coasts. is/are correct?
A. 1 only
78. Consider the following statements: B. 2 and 3 only
1. The pressure gradient is weak where the C. 3 only
isobars are close to each other and is strong D. 1 and 2 only
where the isobars are apart.
Answer: D
2. The rate of change of pressure with respect
to distance is the pressure gradient. Sedimentary rocks are formed by lithification
― consolidation and compaction of
Which of the above statements is/are correct? sediments.
A. 1 only Hence, they are layered or stratified
B. 2 only of varying thickness. Example: sandstone,
shale etc.
C. 1 and 2 both
Sediments are a result of
D. None of the above.
denudation (weathering and erosion) of all
Answer: B types of rocks.
Explanation: These types of rocks cover 75 per cent of the
Statement 1 is incorrect: The pressure gradient is earth’s crust but volumetrically occupy only 5
strong where the isobars are close to each other and per cent (because they are available only in
is weak where the isobars are apart. the upper part of the crust).
Statement 2 is correct” • Ice deposited sedimentary rocks
● The difference in atmospheric pressure are called till or tillite. Wind-
produces a force. deposited sediments are called loess.

● The rate of change of pressure with • Statement 1 is correct: They are


respect to distance is the pressure stratified ― consist of many layers or
gradient. strata. They hold the most informative
geological records due to the marks
● The pressure gradient is strong where the
left behind by various
isobars are close to each other and is weak
geophysical (weather patterns, wind
where the isobars are apart.
and water flow) and biological
activities (fossils).
79. Consider the following statement • Statement 2 is correct: They are
with reference to Sedimentary Rocks: fossiliferous ― have fossils of plants
1. They are stratified and animals.

2. They are fossiliferous • Statement 3 is incorrect: These rocks


are generally porous and allow water
3. These rocks are non- porous
to percolate through them.
of metamorphic rocks under
80. Consider the following statements high pressure.
with reference to Metamorphism: • Sometimes high pressure
1. A magmatic intrusion causing is accompanied by high temperatures
thermal metamorphism is responsible and the action of chemically
for the peak of Mount Everest charged water.

2. Under high pressure, granite • The combination of directed


is converted into gneiss pressure and heat is very powerful in
producing metamorphism because it
Which of the above is/are correct?
leads to more or less complete
A. 1 Only recrystallisation of rocks and the
B. 2 Only production of new structures. This is
known as dynamo thermal
C. Both 1 and 2 metamorphism.
D. Neither 1 nor 2

81. Depending upon the mode of formation,


Statement 1 is correct: A magmatic consider the following examples
intrusion causing thermal metamorphism of sedimentary rocks?
is responsible for the peak of Mount
Sedimentary
Everest consisting of metamorphosed Mode of formation
Rocks
limestone.
Thermal Metamorphism 1. Mechanically
Sandstone
• The change of form or re- formed
crystallisation of minerals
2. Organically
of sedimentary and igneous rocks Potash
formed
under the influence of high
temperatures is known as thermal 3. Chemically
Geyserite
metamorphism. formed
• As a result of thermal
Which of the above is/are correct?
metamorphism, sandstone changes
into quartzite and limestone into A. 1 only
marble. B. 2 and 3 only
• Statement 2 is correct: Under high C. 1 and 3 only
pressure, granite is converted into
D. 2 only
gneiss; clay and shale are transformed
into schist.
Dynamic Metamorphism
• This refers to the formation
Statement 2 is correct: The Alpide
earthquake belt (mid Continental belt)
extends from Java to Sumatra through the
Himalayas, the Mediterranean, and out into
the Atlantic. This belt accounts for about 17
percent of the world's largest earthquakes,
including some of the most destructive.
82. Consider the following Statement 3 is correct: The third
statement regarding Distribution of prominent belt follows the submerged mid-
Earthquakes: Atlantic Ridge. The ridge marks where two
1. Circum-Pacific seismic belt is tectonic plates are spreading apart (a
found along the rim of the Pacific divergent plate boundary). Most of the mid-
Ocean Atlantic Ridge is deep underwater and far
from human development.
2. Alpide earthquake belt extends
from Java to Sumatra
3. The third prominent belt follows 83. Consider the following statements:
the submerged mid-Atlantic Ridge. 1. Surface Waves are also called
Which of the statements given above long period waves because of their
is/are correct? long wavelength.

A. 1 only 2. Love waves are the fastest


surface wave and moves the ground
B. 1 and 3 only
from side-to-side.
C. 3 only
3. Most of the shaking and damage
D. 1,2 and 3 from an earthquake is due to the
Rayleigh wave.

Statement 1 is correct: The world's Which of the statements given above


greatest earthquake belt, the circum-Pacific is/are correct?
seismic belt, is found along the rim of the A. 1 and 2 only
Pacific Ocean, where about 81 percent of our B. 2 and 3 only
planet's largest earthquakes occur. It has
earned the nickname "Ring of Fire". The belt C. 1 and 3 only
exists along boundaries of tectonic plates, D. 1,2 and 3
where plates of mostly oceanic crust are
sinking (or subducting) beneath another
plate. Statement 1 is correct: Surface Waves are
also called long period waves because of their
Earthquakes in these subduction zones are
long wavelength.
caused by slip between plates and rupture
within plates.
Statement 2 is correct: Love waves are • It’s the fastest surface wave and
the fastest surface wave and moves the moves the ground from side-to-side.
ground from side-to-side. Rayleigh waves
Statement 3 is correct: Most of the shaking • A Rayleigh wave rolls along
and damage from an earthquake is due to the ground just like a wave rolls across
the Rayleigh wave. a lake or an ocean.
Additional Info • Because it rolls, it moves the ground
Surface waves (L-Waves) up and down and side-to-side in the
• The body waves interact with same direction that the wave is
the surface rocks and generate new set moving.
of waves called surface waves (long or • Most of the shaking and damage
L-waves). These waves move only from an earthquake is due to the
along the surface. Rayleigh wave.
• Surface Waves are also called
long period waves because of their
84. Consider the following statements:
long wavelength.
1. The trembling on the earth’s
• They are low–frequency surface caused due to P-waves is from
transverse waves (shear waves).
side to side (horizontal).
• They develop in the 2. S-waves are of high frequency
immediate neighbourhood of the and possess slightly higher
epicentre and affect only the surface of destructive power compared to P-
the earth and die out at smaller depth. waves.
• They lose energy more slowly 3. The shadow zone of P-waves is
with distance than the body waves much larger than that of S-waves.
because they travel only across the
surface unlike the body waves which Which of the statements given above
travel in is/are correct?
all directions. A. 1 only
• Particle motion of surface B. 2 and 3 only
waves (amplitude) is larger than that C. 2 only
of body waves, so surface waves are
the most destructive among the D. 1 and 3 only
earthquake waves.
• They are slowest among
the earthquake waves and are
recorded last on the seismograph.
Love waves
86. Which of the following pairs of
volcanic mountains and their location is
incorrectly matched?
A. Mt. Merapi - Sumatra
B. Mt. Mayon- Tanzania
85. Consider the following statements C. Mt. Agung - Bali
with reference to igneous rocks:
D. Mt. Cotopaxi - Ecuador
1. Igneous rocks are also known
as primary rocks
• Mt. Kilimanjaro(Tanzania)
2. If magma cools slowly at great
depths, mineral grains formed in the • Mt. Fuji (Japan)
rocks may be very small. • Mt. Merapi (Sumatra)
3. The Deccan traps in the • Mt. Mayon (Philippines)
Indian peninsular region are of basaltic
origin. • Mt. Agung (Bali)

Which of the statements given above • Mt. Cotopaxi (Ecuador)


is/are correct?
A. 1 only 87. Consider the following pairs
B. 1 and 3 only of volcanoes and their characteristics:

C. 2 only 1. Mid-Ocean Ridge Volcanoes:


The central portion of this
D. 2 and 3 only ridge experiences frequent eruptions.
2. Caldera: They are characterized by
Statement 1 is correct: Igneous rocks the eruption of a cooler and more
are primary rocks, and other rocks form viscous lavas than basalt.
from these rocks.
3. Composite Volcanoes: These are
Statement 2 is incorrect: If magma the most explosive of the
cools slowly at great depths, mineral grains earth’s volcanoes.
formed in the rocks may be very large. Such
4. Cinder Cone Volcanoes:
rocks are called intrusive rocks or plutonic
These volcanoes consist almost
rocks (e.g. Granite). These rocks appear on the
entirely of loose, grainy cinders and
surface only after being uplifted and
almost no lava.
denuded.
Which of the pairs given above
Statement 3 is correct: The Deccan traps in is/are correctly matched?
the Indian peninsular region is of basaltic
origin. A. 1 and 2 only
B. 3 and 4 only a composite volcano or a stratovolcano
(divided into layers).
C. 1 and 4 only
Statement 4 is correct: Cinder
D. 2 and 3 only
Cone Volcanoes: These volcanoes consist
almost entirely of loose, grainy cinders and
Statement 1 is correct: Mid-Ocean Ridge almost no lava. A cinder cone is a steep
Volcanoes: The central portion of this circular or ovalshaped hill of loose pyroclastic
ridge experiences frequent eruptions. The fragments that have been built around a
system of mid-ocean ridges stretches for more volcanic vent.
than 70,000 km across all the ocean basins.
The lava is basaltic (less silica and hence less
viscous) and causes the spreading of the 88. Big stars, with a mass of at least eight
times the mass of our Sun, die in a
seafloor.
spectacular way. As they run out of fuel,
Statement 2 is incorrect: Caldera: In they suddenly collapse and the outer layers
some volcanoes, the magma chamber below of the star are blasted outward in a huge
the surface may be emptied after explosion. What is this explosion known as?
volcanic eruptions. The volcanic material
A. Magnetar
above thechamber collapses into the empty
magma chamber, and the collapsed surface B. Neutron Star
appears like a large cauldron-like hollow (tub C. Supernova
shaped) called the caldera. When water from
D. Black Hole
rain or melted snow gets accumulated in the
caldera, it becomes a caldera lake (in general, Answer: C
the caldera lakes are also called crater lakes). Explanation:
Due to their unstable environments, some
● Big stars, with a mass at least eight
crater lakes exist only intermittently. Caldera
times the mass of our Sun, die in a
lakes, in contrast, can be quite large and long-
spectacular way. As they run out of
lasting. For example, Lake Toba
fuel, they suddenly collapse, then the
(Indonesia) formed after its supervolcanic
outer layers of the star are blasted
eruption around 75,000 years ago. It is the
outward in a huge explosion known as
largest crater lake in the world.
a supernova.
Statement 3 is incorrect:
● The energy released by a supernova is
Composite Volcanoes: They are conical or
as much as the energy radiated by the
central type volcanic landforms. Along with
Sun during its entire lifetime. One
andesitic lava, large quantities of pyroclastic
supernova can outshine a galaxy
material and ashes find their way to the
containing billions of stars.
surface. Andesitic lava along with pyroclastic
material accumulates in the vicinity of the ● Supernovas are rare events- none have
vent openings leading to the formation of been seen in our galaxy since the
layers, and this makes the mounts appear as invention of the telescope.
Option a is incorrect: A magnetar is a neutron
star, “the crushed, city-size remains of a star
many times more massive than our Sun.”
Option b is incorrect: Neutron stars are small,
only about 6 miles (10 km) across, yet they are
heavier than the Sun. Neutron stars are
covered by an iron crust, 10 billion times
stronger than steel. Inside, they contain a
liquid sea of neutrons- the debris from atoms
crushed by a supernova explosion.

Option d is incorrect: A black hole is an object


in space that is so dense and has such strong
gravity that no matter or light can escape its 90. Consider the following pairs of Space
pull. Because no light can escape, it is black Mission and their Objectives:
and invisible. Objectives/
Space Mission
Description
89. Consider the following Layers of Sun:
1. Extreme
1. Radiative Zone Ultraviolet High- Heliophysics
2. Convective Zone Throughput Missions to
Spectroscopic Explore Sun and
3. Photosphere
Telescope (EUVST) Earth’s Aurora.
4. Chromosphere mission
5. Corona
Aim is to study
Arrange the following layers as we move
how the solar wind
from center of sun to its outer part?
2. PUNCH mission and transients
A. 1-2-3-4-5 evolve through the
B. 2-1-4-3-5 inner solar system.

C. 5-4-3-2-1 Aim of the mission


D. 2-1-5-3-4 is to observe the
Answer: A 3. Parker Solar Probe outer corona of the
Sun.

Which of the above is/are correct?


A. 1 and 2 only
B. 2 and 3 only
C. 1 and 3 only Answer: D
D. 1,2 and 3 Explanation
Answer: D Option a is correct: Artemis: NASA: Artemis
is NASA’s moon mission under which it will
Explanation:
land the first woman and the next man on
Option 1 is correct: Extreme Ultraviolet High- Moon by 2024
Throughput Spectroscopic Telescope
Option b is correct: Chang’e 5: China: Chang'e
(EUVST) mission - NASA- Heliophysics
is the fifth lunar exploration mission of the
Missions to Explore Sun and Earth’s Aurora
Chinese Lunar Exploration Program, and
Option 2 is correct: PUNCH mission China's first lunar sample-return mission
(Polarimeter to Unify the Corona and
• The mission recently returned to Earth,
Heliosphere) - NASA- PUNCH mission will
loaded with moon rocks.
study this transition and how the solar wind
and transients evolve through the inner solar • The mission made China the third
system. Launch date - 2023 country to return samples from the
Moon after the United States and the
Option 3 is correct: Parker Solar Probe-
Soviet Union
NASA- The Parker Solar Probe (abbreviated
PSP; previously Solar Probe, Solar Probe Plus Option c is correct: Volatiles Investigating
or Solar Probe+). It is a NASA space probe Polar Exploration Rover (VIPER): NASA: It is
launched in 2018 with the mission of making NASA’s first mobile robot to be sent to the
observations of the outer corona of the Sun. Moon in late 2023 in search of ice and other
resources on and below the lunar surface. It is
part of NASA’s Artemis programme
91. Consider the following pairs of mission
on moon and the agencies involved. Which Option d is incorrect: Stratospheric
of the following is incorrect? Observatory for Infrared Astronomy
(SOFIA): NASA and German Aerospace
Agencies Center: SOFIA has detected water molecules
Missions on moon
involved (H2O) in Clavius Crater located in the Moon’s
southern hemisphere
Artemis NASA

Chang’e 5 China 92. Consider the following statements with


reference to Continental Drift Theory:
Volatiles Investigating
Polar Exploration Rover NASA 1. Continental Drift Theory explains the
(VIPER) movement of Lithospheric plates that
include both continents and oceans.
Stratospheric
NASA and 2. According to Plate tectonics,
Observatory for Infrared
JXA convection currents in the mantle is
Astronomy (SOFIA)
responsible for the movement of the 1. Arthur Holmes had discussed the
crustal plates. possibility of convection currents
operating in the mantle portion.
3. Continental Drift Theory helped in the
evolution of convection current theory 2. Wherever rising limbs of these
and seafloor spreading theory currents meet, trenches are formed on
Which of the statements given above is/are the sea floor and wherever the failing
correct? limbs meet, oceanic ridges are formed.

A. 1 only Which of the statements given above is/are


correct?
B. 1 and 3 only
A. 1 only
C. 2 only
B. 2 only
D. 2 and 3 only
C. Both 1 and 2
Answer: D
D. Neither 1 nor 2
Explanation
Answer: A
• Continental Drift Theory was put
Explanation
forward by the German scientist
Alfred Wegner in 1915. ● In the 1930s, Arthur Holmes proposed
the Convectional Current Theory.
• According to the Continental Drift
Convection Current Theory is the soul
Theory, part of the crust is capable of
of Seafloor Spreading Theory.
horizontal movement round the globe
causing the continents to slowly ● According to this theory, the extreme
change their positions in relation to heat generated by radioactive
one another. chemicals in the mantle (100-2900 km
beneath the earth's surface) seeks a
Statement 1 is incorrect: Continental Drift
way to escape and causes convection
Theory explains the movement of Continents
currents to emerge in the mantle.
only and also it doesn’t talk about plates
concept. ● Under the solid lithosphere, the fluid
asthenosphere circulates as convection
Statement 2 is correct: According to Plate
currents.
tectonics, convection currents in the mantle is
responsible for the movement of the crustal ● Divergent convective currents pull
plates. crustal blocks away from each other in
opposite directions, forming seas and
Statement 3 is correct: Continental Drift
oceans. East Africa's lakes system, the
Theory helped in the evolution of convection
Mid-Atlantic Ridges, and so on are
current theory and seafloor spreading theory
examples.
● Convergent convective currents
93. Consider the following statements with generate subsidence in the crustal
reference to Convection Current Theory: zones, resulting in geosynclines and
sea closure, resulting in the Answer: D
construction of island arcs, mountains, Explanation
and other features.
● The Caledonian and Hercynian
● Convection currents run through the mountains of Europe and the
mantle's molten rock, acting as a Appalachians of USA seem to be one
conveyor belt for the plates. continuous series.
● The total change in the lithospheric
materials is constant since these
currents are closed in nature.
Statement 1 is correct: Arthur Holmes in
1930s discussed the possibility of convection
currents in the mantle. These currents are
generated due to radioactive elements
causing thermal differences in the mantle.
Statement 2 is incorrect: Wherever rising
limbs of these currents meet, oceanic ridges
are formed on the sea floor and wherever the Continuous Very Old Fold Mountain Chain
failing limbs meet, trenches are formed.
1. Polar wandering (Shifting of Poles)
● The position of the poles constantly
94. Which of the following is/are evidences drifted (due to plate tectonics).
of Continental Drift Theory? 2. Botanical Evidence
1. Rich placer deposits of gold are found
on the Ghana coast (West Africa) but
the source (gold-bearing veins) are in
Brazil
2. The Caledonian and Hercynian
mountains of Europe and the
Appalachians of USA seem to be one
continuous series.
3. Skeletons of Mesosaurus have been
found only in South Africa and Brazil.
Select the correct code:
A. 1 and 2 only
B. 2 and 3 only
C. 1 and 3 only
D. 1,2 and 3
4. Tillite Deposits
● Tillite deposits are sedimentary rocks
formed out of deposits of glaciers.
● The Gondwana system of sediments
are found in India, Africa, Falkland
Island, Madagascar, Antarctica and
Australia (all were previously part of
Gondwana).
● Overall resemblance demonstrates
that these landmasses had remarkably
similar histories.
Distribution of Fossils across the 5. Placer Deposits
Gondwanaland ● Rich placer deposits of gold are found
● Presence of glossopteris vegetation in on the Ghana coast (West Africa) but
Carboniferous rocks of India, the source (gold-bearing veins) are in
Australia, South Africa, Falkland Brazil, and it is obvious that the gold
Islands (Overseas territory of UK), deposits of Ghana are derived from the
Antarctica, etc. (all split from the same Brazil plateau when the two continents
landmass called Gondwana) can be lay side by side.

● explained from the fact that parts were


linked in the past. 95. According to the theory of plate tectonics,
Distribution of Fossils which of the following is not one of the
seven major plates?
● The observations that Lemurs occur in
India, Madagascar and Africa led some A. North American plate
to consider a contiguous landmass B. South American plate
“Lemuria” linking these three
C. Africa with the eastern Atlantic floor
landmasses.
plate
● Mesosaurus was a small reptile
D. Arabian plate
adapted to shallow brackish water.
The skeletons of these are found only Answer: D
in South Africa and Brazil. The two Explanation
localities presently are 4,800 km apart
Major Tectonic Plates
with an ocean in between them.
● Antarctica and the surrounding
3. Rocks of Same Age Across the Oceans
oceanic plate
● The belt of ancient rocks of 2,000
million years from Brazil coast ● North American plate
matches with those from western ● South American plate
Africa.
● Pacific plate
● India-Australia-New Zealand plate the East Pacific Rise in the South Pacific
(about 3,400 km west of Chile), has the fastest
● Africa with the eastern Atlantic floor
rate (more than 15 cm/year).
plate
● Eurasia and the adjacent oceanic plate
97. Consider the following statements with
reference to Plate Tectonics:
96. Consider the following statements:
1. The oceanic plates contain mainly the
1. Convection currents in the mantle that Sialic crust and are relatively thinner
are generated due to thermal gradients
2. The continental plates contain Simatic
is the force for plate movement
material and are relatively thicker.
2. The Arctic Ridge has the fastest rate of
3. The Indian plate includes Peninsular
plate movement, and the East Pacific
India and the Australian continental
Rise in the South Pacific, has the
portions.
slowest rate of plate movement.
Which of the statements given above is/are
Which of the statements given above is/are
correct?
correct?
A. 1 only
A. 1 only
B. 2 and 3 only
B. 2 only
C. 3 only
C. Both 1 and 2
D. 1, 2 and 3
D. Neither 1 nor 2
Answer: C
Answer: A
Explanation
Explanation
● It was from the continental drift
Convectional Current Theory was given by
theory, convection current theory and
Arthur Holmes in the 1930s. It discussed the the theory of seafloor spreading, the
possibility of convection currents operating in theory of Plate Tectonics was
the mantle portion These currents are formulated.
generated due to radioactive elements
● In 1967, McKenzie and Parker
causing thermal differences in the mantle
suggested the theory of plate tectonics.
portion. Holmes argued that there exists a
Morgan later outlined the theory in
system of such currents in the entire mantle
1968.
portion.
● According to the theory of plate
Statement 1 is correct: Convection currents in
tectonics, the earth’s lithosphere is
the mantle that are generated due to thermal
broken into distinct plates which are
gradients is the force for plate movement
floating on a ductile layer called
Statement 2 is incorrect: The Arctic Ridge has asthenosphere (upper part of the
the slowest rate (less than 2.5 cm/year), and mantle).
● Plates move horizontally over the 2. Wind
asthenosphere as rigid units. 3. Gravity
● The lithosphere includes the crust and 4. Coriolis force
top mantle with its thickness range
varying between 5-100 km in oceanic 5. Temperature difference

● parts and about 200 km in the 6. Salinity difference


continental areas. Select the correct code:
● The oceanic plates contain mainly the A. 1 and 2 only
Simatic crust and are relatively
B. 3, 4 and 5 only
thinner, while the continental plates
contain Sialic material and are C. 5 and 6 only
relatively thicker. D. 1, 2, 3 and 4 only
● Lithospheric plates (tectonic plates)
vary from minor plates to major plates,
The primary forces that influence the
continental plates (Arabian plate) to currents are:
oceanic plates (Pacific plate),
sometimes a combination of both • heating by solar energy;
continental and oceanic plates (Indo- • wind;
Australian plate).
• gravity;
● The movement of these crustal plates
• Coriolis force.
(due to convection currents in the
mantle) causes the formation of Additional info:
various landforms and is the principal • What is Ocean Current? It is a
cause of all earth movements. horizontal movement of seawater that
Statement 1 is incorrect: The oceanic plates is produced by gravity, wind, and
contain mainly the Simatic crust and are water density. Ocean currents play an
relatively thinner important role in the determination of
climates of coastal regions.
Statement 2 is incorrect: The continental
plates contain Sialic material and are • Ocean currents are influenced by two
relatively thicker. types of forces namely: primary forces
that initiate the movement of water;
Statement 3 is correct: The Indian plate
secondary forces that influence the
includes Peninsular India and the Australian
currents to flow.
continental portions.
• The secondary forces that influence
the currents are:
98. Which of the following is/are primary
o Temperature difference;
forces that influence the currents are?
o Salinity difference
1. Heating by solar energy
99. Consider the following statements: 2. Diurnal and annual ranges of
1. Seamount is a mountain that reaches temperature cease after the depth of 5
the surface of the ocean. fathoms and 100 fathoms respectively.

2. Seamounts are volcanic in origin. 3. High temperatures at greater depths


are recorded in Sargasso Sea.
3. Guyots are flat topped mountains
Which of the above is/are correct?
4. Seamounts and guyots are very
A. 1 only
common in the Atlantic Ocean
Which of the above is/are correct? B. 2 only

A. 1 and 2 only C. 1 and 3 only

B. 3 and 4 only D. 2 and 3 only

C. 1 and 4 only
Statement 1 is incorrect: Though the sea
D. 2 and 3 only
temperature decreases with increasing depth,
the rate of decrease of temperature is not
Statement 1 is incorrect: Seamount is a uniform. The change in sea temperature
mountain with pointed summits, rising from below the depth of 2000m is negligible.
the seafloor that does not reach the surface of Statement 2 is correct: Diurnal and annual
the ocean. ranges of temperature cease after the depth of
Statement 2 is correct: Seamounts are volcanic 5 fathoms (30 feet) and 100 fathoms (600 feet)
in origin. These can be 3,000-4,500 m tall. The respectively.
emperor seamount, an extension of the Statement 3 is correct: High temperature at
Hawaiian Islands [Hotspot] in the Pacific greater depths is recorded in Sargasso Sea.
Ocean, is a good example.
Additional info:
Statement 3 is correct: Guyots are flat topped
The following are the salient features of
mountains (seamounts).
vertical distribution of temperature of ocean
Statement 4 is incorrect: Seamounts and water:
guyots are very common in the Pacific Ocean
• Though the sea temperature decreases
where they are estimated to number around
with increasing depth but the rate of
10,000.
decrease of temperature is not
uniform. The change in sea
100. Consider the following salient features temperature below the depth of 2000m
of vertical distribution of temperature of is negligible.
ocean water? • Diurnal and annual ranges of
1. The sea temperature decreases with temperature cease after the depth of 5
increasing depth but the rate of fathoms (30 feet) and 100 fathoms (600
decrease of temperature is uniform. feet) respectively.
• The rate of decrease of temperature structure of ocean water.
with increasing depth from equator
towards the poles is not uniform.
• Though the surface temperature of the
seas decreases from equator towards
the poles but the temperature at the
ocean bottoms is uniform from the
equator towards the pole, which
means that the rate of decrease of tem-
perature with increasing depth is more
rapid near the equator than towards
the poles.
• The areas from where sea surface
water is driven away by offshore
winds resulting into upwelling of
water from below record low
temperature at sea surface and thus the
rate of decrease of temperature with
increasing depth becomes low.
• Contrary to this the areas where there
is pilling of sea water because of
onshore winds, record relatively high
temperature at sea surface and thus the
rate of decrease of temperature with
increasing depth becomes rapid.
• In some areas high temperature is
recorded at greater depths e.g., in
Sargasso Sea, Red Sea, Mediterranean
Sea, Sulu Sea etc. The Mediterranean
Sea records 24.4°C at the depth of
1,829m whereas the Indian Ocean has
only 1.1°C temperature at the same
depth. Such anomalous conditions are
noticed in the enclosed seas of low
latitudes. The enclosed seas of high
latitudes register inversion of tempera-
ture i.e., the temperature of sea surface
is lower than the temperature below.
• There is a clear-cut layered thermal

You might also like